Foro de preguntas y respuestas de matemáticas, de cualquier nivel. Cuánto más interesantes, divertidas o intrépidas, mejor.
Aviso: Te invitamos a conocer la página de Facebook de la UCIM

Ganas puntos al hacer preguntas, contestarlas y, sobre todo, si tu respuesta es seleccionada como la mejor.
Registrate como usuario para participar en el foro. También puedes utilizar tu identidad de FB Utiliza el botón azul para ingresar (si usas tu identidad de FB y estás logeado en FB, automáticamente te reconoce).

El irracional tiene una página en FB. El Irracional






+2 votos
Hola, existe una función $f$ que tiene como dominio las parejas de reales $(x,y)$ con $x<y$ a los reales que cumple que $f(a,b)\leq f(c,d)$ si y solo si $a<f(c,d)$ y $f(a,b)<d$?

 

Este es el avance que llevo:

 

Tomando $a=c$ y $b=d$ llegamos a que $a<f(a,b)<b$.

 

Usando esto tomemos $(a,b)$ y tomemos $f(a,b)=z$. Ahora tomemos $x,y$ tal que $a<x<z<y<b$.

 

Podemos mostar que $f(x,y)\leq z$ pues $f(x,y)<y<b$ y $x<z$.

 

También podemos probar $f(x,y)\geq z$ pues $y>z$ y $f(x,y)>x>a$.

 

Entonces $f(x,y)=z$. Lo cual me hace creer que la funcion no existe, pero no he podido encontrar una contradiccion contundente.
por (2,4m puntos) en Conjeturas
editado por
Tienes un error al mero principio. Si $a<f(a,b)<b$ se tendría que $f(a,b)<f(a,b)$. Por lo tanto $f(a,b)\leq a$ o bien $b\leq f(a,b)$. Posiblemente querías intercambiar desigualdades estrictas con no estrictas.
A sí es cierto, pero el error lo hice en el enunciado afortunadamente y no en la solución, deja lo edito.

1 Respuesta

+3 votos

Para reales $a<b$ y entero $k$ considera el conjunto $(a,b)\cap\mathbb{Z}\cdot 2^k$. Es facil ver que existe un entero minimal tal que este conjunto consiste de un solo elemento. Definimos $g(a,b)$ como este entero y $f(a,b)$ como el unico punto de este conjunto.

Por definicion tenemos $a<f(a,b)<b$ y esto implica la direccion $\Rightarrow$ de la propiedad deseada para la funcion $f$.

Ahora supongamos que $a<f(c,d)<f(a,b)<d$. Esto implica 

$a<f(c,d)<f(a,b)<b$ y

$c<f(c,d)<f(a,b)<b$.

Si $k=\min\{g(c,d),g(a,b)\}$ entonces $f(c,d),f(a,b) \in (a,b)\cap(c,d)\cap\mathbb{Z}\cdot 2^k$ lo cual contradice la definicionde la funcion $g$. Por lo que concluimos que $a<f(c,d)$ y $f(a,b)<d$ implica $f(a,b)\leq f(c,d)$.

 
por (17,3m puntos)
editado por
Licencia Creative Commons
Este obra está bajo una Licencia Creative Commons Atribución-NoComercial-CompartirIgual 2.5 México.

powered by UCIM  -  Aviso de privacidad

...